Un équivalent

Bonjour,

Soient $X_1,...,X_n$ indépendantes de même loi géométrique à paramètre $1/n$, et $X=\max\big(X_1,...,X_n\big)$. Une expression de l'espérance de $X$ est
$$a_n = \sum_{i=1}^n\binom{n}{i}(-1)^{i+1}\frac{1}{1-\big(\frac{n-1}{n}\big)^i}$$
Est-il possible de trouver un équivalent simple de $a_n$ quand $n\rightarrow +\infty$?

Ma question vient de ce fil, où j'ai cherché (par erreur) un équivalent de cette expression.

EDIT: J'ai corrigé l'expression suite à la remarque de JLT.

Réponses

  • En approximant $X_k$ par une loi exponentielle de paramêtre $\theta\sim\frac{1}{n}$, en calculant la densité d'un max de variable exponentielles puis en estimant l'espérance, j'ai obtenu l'équivalent $n\ln n$. J'ai fait vite donc les erreurs de calculs sont envisageables
  • @Namiswan: j'ai commencé comme toi, mais je ne vois pas bien quoi faire de

    $\int_0^{+\infty} 1-(1-e^{-t})^n \ dt$.
  • Un changement de variables $u=(1-e^{-t})$ permet de conclure.
  • Si je pars de l'expression donnée de $a_n$, je trouve par le calcul et par des essais numériques que $a_n$ tend vers $1/e$. Est-ce que je délire ?

    Edit : le message initial a été modifié.
  • Ah, merci BobbyJoe !
  • @JLT: Oups, il fallait remplacer le $1/n$ par $(n-1)/n$ dans l'expression, merci beaucoup. Le code suivant (sur R) montre qu'avec cette expression, on a bien divergence
    compute_expectation <- function(n) {
      vec_binom <- choose(n,1:n)
      vec_1 <- (-1)^((1:n)+1)
      vec_n <- 1/(1-(((n-1)/n)^(1:n)))
      a <- sum(vec_binom * vec_1 * vec_n)
      return(a)
    }
    
    vec_n <- 2:40
    vec_a <- vapply(vec_n, compute_expectation, numeric(1))
    plot(vec_n, vec_a)
    
    vec_equiv <- vec_n * log(vec_n)
    plot(vec_n, vec_a / vec_equiv)
    
  • Merci beaucoup aléa, Namiswan et BobbyJoe, je retiens l'idée de passer à une loi continue pour faciliter les calculs.
  • En relisant le message de JLT, je suis perdu. Si $X_i$ suit une loi de paramètre $p\in ]0,1[$, je trouve
    $$a_n(p)=\mathbb{E}[X]=\sum_{i=1}^n\binom{n}{i}(-1)^{i+1}\frac{1}{1-(1-p)^i}$$
    Par ailleurs, on peut considérer que $X_i=\lceil Y_i\rceil$ où $Y_i$ suit une loi exponentielle de paramètre $p$, i.e., (EDIT: c'est faux, cf. messages ci-dessous)
    $$Y_i\leq X_i < Y_i + 1$$
    En notant $Y=\max\big(Y_1,...,Y_n\big)$, il vient
    $$Y\leq X< Y + 1$$
    Et donc
    $$E(Y)\leq E(X)<E(Y)+1$$
    Or l'espérance de $Y$ est donnée par
    \begin{eqnarray*}
    \int_0^{+\infty}P(Y>x)dx&=&\int_0^{+\infty}1-(1-e^{-px})^ndx \\
    &=&\frac{1}{p}\int_0^{+\infty}1-(1-e^{-t})^ndt \\
    &=&\frac{1}{p}\int_0^1 \frac{1-u^n}{1-u}du \\
    &=&\frac{1}{p}\sum_{i=1}^n\frac{1}{i}
    \end{eqnarray*}
    C'est-à-dire
    $$\frac{1}{p}\sum_{i=1}^n\frac{1}{i}\leq a_n(p)<\frac{1}{p}\sum_{i=1}^n\frac{1}{i}+1$$
    D'où, en prenant $p_n=1/n$, l'équivalent $a_n(p_n)\sim n\ln n$. En prenant $q_n=(n-1)/n$, on retrouve l'expression étudiée par JLT
    $$a_n(q_n) = \sum_{i=1}^n\binom{n}{i}(-1)^{i+1}\frac{1}{1-\frac{1}{n^i}}$$
    Numériquement, $a_n(q_n)\rightarrow 1/e$ (je n'ai pas vérifié la valeur, mais un plot des 50 premières valeurs suggère que la suite converge) or d'après les calculs précédents, je trouve $a_n(q_n)\sim\ln n$... :-S
  • C'est parce que le paramètre de la loi exponentielle n'est pas $p$, mais c'est le réel $\lambda$ tel que $1-p=e^{-\lambda}$. Lorsque $p=1-1/n$ on a $\lambda=\ln n$, donc tu trouves que $a_n$ est encadré entre deux constantes, ce qui est cohérent avec $a_n\to 1/e$.
  • @JLT: Merci beaucoup c'est très clair. Pour que l'encadrement fonctionne, il suffit donc d'avoir $\lambda_n = o(\ln n)$, correct ? Aurais-tu une indication pour comment procéder quand $p_n = 1-1/n$ ?
  • Oui c'est bien ça. Pour $p_n=1-1/n$ on a $a_n=1+\sum_{i=1}^n (-1)^{i+1}\binom{n}{i}(\frac{1}{1-\frac{1}{n^i}}-1)$, et pour tout $i$ le terme $\binom{n}{i}(\frac{1}{1-\frac{1}{n^i}}-1)$ tend vers $\frac{1}{i!}$ donc ça devrait aboutir, mais je n'ai pas vérifié les détails.

    (Je crois que la limite est plutôt $2-1/e$, j'ai fait une faute de signe dans mon brouillon précédent semble-t-il).
  • Merci. Avec l'indication, voici une preuve un peu calculatoire
    \begin{eqnarray*}
    \left| \sum_{i=1}^n\left[(-1)^{i+1}\binom{n}{i}\left(\frac{1}{1-\frac{1}{n^i}}-1\right)-\frac{(-1)^{i+1}}{i!}\right]\right| &\leq& \sum_{i=1}^n\frac{1}{i!}\left|\frac{n(n-1)...(n-i+1)}{n^i-1}-1\right| \\
    &=& \frac{1}{n-1} + \sum_{i=2}^n\frac{1}{i!}\left(1-\frac{n(n-1)...(n-i+1)}{n^i-1}\right) \\
    &\leq& \frac{1}{n-1}+\sum_{i=2}^n\frac{1}{i!n^i}\left(n^i-n(n-1)...(n-i+1)\right) \\
    &=&\frac{1}{n-1}-\sum_{i=2}^n\frac{1}{i!}\sum_{j=0}^{i-1}a^{(i)}_jn^{j-i}
    \end{eqnarray*}
    Où $a_0^{(i)},...,a_i^{(i)}$ les coefficients sont les coefficients de $n(n-1)...(n-i+1)$, qui est un polynôme en $n$ de degré $i$. On remarque que $a^{(i)}_i = 1$ et $a^{(i)}_{i-1}=-\frac{i(i-1)}{2}$. Alors
    \begin{eqnarray*}
    \left|\frac{1}{i!}\sum_{j=0}^{i-1}a^{(i)}_jn^{j-i}\right|&\leq&\left|\frac{a^{(i)}_{i-1}}{i!}n^{-1}\right| +n^{-2}\frac{1}{i!}\sum_{j=0}^{i-2}\left|a^{(i)}_jn^{j+2-i}\right| \\ &\leq& \frac{i(i-1)}{i!}n^{-1} + n^{-2}\sum_{j=0}^{i-2}\left|\frac{a^{(i)}_j}{i!}\right| \\
    &\leq& \frac{i(i-1)}{i!}n^{-1} + n^{-2}\sum_{j=0}^{i}\left|\frac{a^{(i)}_j}{i!}\right| \\
    &=& \frac{i(i-1)}{i!}n^{-1} + n^{-2}
    \end{eqnarray*} où on peut montrer $\sum_{j=0}^{i}\left|\frac{a^{(i)}_j}{i!}\right| =1$ par récurrence. D'où
    \begin{eqnarray*}
    \left|\sum_{i=2}^n\frac{1}{i!}\sum_{j=0}^{i-1}a^{(i)}_jn^{j-i}\right|\leq\sum_{i=2}^n\left|\frac{1}{i!}\sum_{j=0}^{i-1}a^{(i)}_jn^{j-i}\right|\leq\frac{1}{n}\sum_{i=2}^n\frac{i(i-1)}{i!}+\frac{1}{n}\longrightarrow 0
    \end{eqnarray*}
    En recollant les morceaux je trouve bien
    $$\lim a_n = 2-1/e$$

    EDIT: J'ai corrigé la fin de la preuve.
  • J'ai eu un peu de mal à comprendre ce fil car il y a deux questions différentes qui y sont traitées.

    Pourquoi utiliser cette expression de l'espérance qui a l'avantage d'être une somme finie mais qui est compliquée à étudier en $+\infty$ ?

    L'étude en $+\infty$ est plus simple sous la forme $a_n=\displaystyle\sum_{k=0}^{+\infty}\mathbb{P}(X>k)=\displaystyle\sum_{k=0}^{+\infty}(1-(1-q^k)^n)$ pour les $X_k$ de loi géométrique de paramètre $p$ (avec $q=1-p$).

    Dans le cas $p=\dfrac1n$, $q=1-\dfrac1n$, on compare à l'intégrale $\displaystyle\int_{0}^{+\infty}(1-(1-(1-1/n)^t)^n)\;dt$ qui se calcule par $x=1-(1-1/n)^t$.
    Elle est égale à $-\dfrac1{\ln(1-1/n)}H_n\sim n\ln n$ qui fournit l'équivalent de $a_n$.

    Dans le cas $p=1-\dfrac1n$, $q=\dfrac1n$, $a_n=1+1-(1-1/n)^n+\displaystyle\sum_{k=2}^{+\infty}u_k$ avec $u_k=1-(1-1/n^k)^n$.
    Avec $1-e^{-x}\leq x$ on a d'abord $u_k\leq -n\ln(1-1/n^k)$.
    Puis avec $-\ln(1-x)\leq2x$ pour $0\leq x\leq 1/2$ on obtient $u_k\leq 2/n^{k-1}$ d'où $\displaystyle\sum_{k=2}^{+\infty}u_k\leq \dfrac2{n-1}$ puis $\lim(a_n)=2-e^{-1}$.
  • @jandri: Merci (tu) Le cas $p=1-1/n$ est beaucoup mieux traité comme ça qu'avec mes polynômes
Connectez-vous ou Inscrivez-vous pour répondre.